Prove uniform continuity of the composition of two uniformly continuous functions. Announcing the arrival of Valued Associate #679: Cesar Manara Planned maintenance scheduled April 17/18, 2019 at 00:00UTC (8:00pm US/Eastern)product of two uniformly continuous functions is uniformly continuousContinuity and uniformly continuous proofProve that $1/f$ is uniformly continuous on …$sqrt x$ is uniformly continuousUniform continuity implies we just need a small enough $delta$?Continuous functions which are Uniformly continuousContinuity of a composition of continuous functionsComposition of Absolutely Continuous FunctionsProof verification of uniform continuityComposition of uniformly continuous functions.

Can I throw a longsword at someone?

Why does tar appear to skip file contents when output file is /dev/null?

Are my PIs rude or am I just being too sensitive?

Can a zero nonce be safely used with AES-GCM if the key is random and never used again?

What is the electric potential inside a point charge?

How to set letter above or below the symbol?

What do you call the holes in a flute?

How to say 'striped' in Latin

How to say that you spent the night with someone, you were only sleeping and nothing else?

Slither Like a Snake

Choo-choo! Word trains

Antler Helmet: Can it work?

Blender game recording at the wrong time

What did Darwin mean by 'squib' here?

I'm having difficulty getting my players to do stuff in a sandbox campaign

Using "nakedly" instead of "with nothing on"

Can a monk deflect thrown melee weapons?

Active filter with series inductor and resistor - do these exist?

Do working physicists consider Newtonian mechanics to be "falsified"?

What do I do if technical issues prevent me from filing my return on time?

Simulating Exploding Dice

What was the last x86 CPU that did not have the x87 floating-point unit built in?

Determine whether f is a function, an injection, a surjection

Who can trigger ship-wide alerts in Star Trek?



Prove uniform continuity of the composition of two uniformly continuous functions.



Announcing the arrival of Valued Associate #679: Cesar Manara
Planned maintenance scheduled April 17/18, 2019 at 00:00UTC (8:00pm US/Eastern)product of two uniformly continuous functions is uniformly continuousContinuity and uniformly continuous proofProve that $1/f$ is uniformly continuous on …$sqrt x$ is uniformly continuousUniform continuity implies we just need a small enough $delta$?Continuous functions which are Uniformly continuousContinuity of a composition of continuous functionsComposition of Absolutely Continuous FunctionsProof verification of uniform continuityComposition of uniformly continuous functions.










0












$begingroup$


Given two functions that are uniformly continuous on $mathbbR$ prove that their composition is uniformly continuous. Here's my attempt.



Proof



Let $f,g$ be our given uniformly continuous functions. We need to show that for any $epsilon >0 $ there exists a $delta$ such that



$$left| (fcirc g)left(xright) - (fcirc g)left(yright) right| < epsilon$$



Since we are given $$left|x' - y' right| < delta_f implies left| fleft(x'right) - fleft(y'right) right| < epsilon$$



and $$left|x - y right| < delta implies left| gleft(xright) - gleft(yright) right| < delta_f$$



defining $g(a) = a'$ for $ain mathbbR$ completes the proof.










share|cite|improve this question











$endgroup$
















    0












    $begingroup$


    Given two functions that are uniformly continuous on $mathbbR$ prove that their composition is uniformly continuous. Here's my attempt.



    Proof



    Let $f,g$ be our given uniformly continuous functions. We need to show that for any $epsilon >0 $ there exists a $delta$ such that



    $$left| (fcirc g)left(xright) - (fcirc g)left(yright) right| < epsilon$$



    Since we are given $$left|x' - y' right| < delta_f implies left| fleft(x'right) - fleft(y'right) right| < epsilon$$



    and $$left|x - y right| < delta implies left| gleft(xright) - gleft(yright) right| < delta_f$$



    defining $g(a) = a'$ for $ain mathbbR$ completes the proof.










    share|cite|improve this question











    $endgroup$














      0












      0








      0





      $begingroup$


      Given two functions that are uniformly continuous on $mathbbR$ prove that their composition is uniformly continuous. Here's my attempt.



      Proof



      Let $f,g$ be our given uniformly continuous functions. We need to show that for any $epsilon >0 $ there exists a $delta$ such that



      $$left| (fcirc g)left(xright) - (fcirc g)left(yright) right| < epsilon$$



      Since we are given $$left|x' - y' right| < delta_f implies left| fleft(x'right) - fleft(y'right) right| < epsilon$$



      and $$left|x - y right| < delta implies left| gleft(xright) - gleft(yright) right| < delta_f$$



      defining $g(a) = a'$ for $ain mathbbR$ completes the proof.










      share|cite|improve this question











      $endgroup$




      Given two functions that are uniformly continuous on $mathbbR$ prove that their composition is uniformly continuous. Here's my attempt.



      Proof



      Let $f,g$ be our given uniformly continuous functions. We need to show that for any $epsilon >0 $ there exists a $delta$ such that



      $$left| (fcirc g)left(xright) - (fcirc g)left(yright) right| < epsilon$$



      Since we are given $$left|x' - y' right| < delta_f implies left| fleft(x'right) - fleft(y'right) right| < epsilon$$



      and $$left|x - y right| < delta implies left| gleft(xright) - gleft(yright) right| < delta_f$$



      defining $g(a) = a'$ for $ain mathbbR$ completes the proof.







      real-analysis uniform-continuity






      share|cite|improve this question















      share|cite|improve this question













      share|cite|improve this question




      share|cite|improve this question








      edited Feb 22 '18 at 4:26









      Eric Wofsey

      193k14221352




      193k14221352










      asked Feb 22 '18 at 4:15









      ZduffZduff

      1,7501021




      1,7501021




















          1 Answer
          1






          active

          oldest

          votes


















          3












          $begingroup$

          This is the right idea but what you have written is nonsense if taken literally. It is extremely important to clearly state the hypotheses and quantifiers in each statement you make. For instance, we are not "given" $$left|x' - y' right| < delta_f implies left| fleft(x'right) - fleft(y'right) right| < epsilon.$$ Rather, we are given that there exists $delta_f>0$ such that for all $x',y'inmathbbR$, this implication is true. Clean up all your statements similarly and you'll have a great proof.






          share|cite|improve this answer









          $endgroup$













            Your Answer








            StackExchange.ready(function()
            var channelOptions =
            tags: "".split(" "),
            id: "69"
            ;
            initTagRenderer("".split(" "), "".split(" "), channelOptions);

            StackExchange.using("externalEditor", function()
            // Have to fire editor after snippets, if snippets enabled
            if (StackExchange.settings.snippets.snippetsEnabled)
            StackExchange.using("snippets", function()
            createEditor();
            );

            else
            createEditor();

            );

            function createEditor()
            StackExchange.prepareEditor(
            heartbeatType: 'answer',
            autoActivateHeartbeat: false,
            convertImagesToLinks: true,
            noModals: true,
            showLowRepImageUploadWarning: true,
            reputationToPostImages: 10,
            bindNavPrevention: true,
            postfix: "",
            imageUploader:
            brandingHtml: "Powered by u003ca class="icon-imgur-white" href="https://imgur.com/"u003eu003c/au003e",
            contentPolicyHtml: "User contributions licensed under u003ca href="https://creativecommons.org/licenses/by-sa/3.0/"u003ecc by-sa 3.0 with attribution requiredu003c/au003e u003ca href="https://stackoverflow.com/legal/content-policy"u003e(content policy)u003c/au003e",
            allowUrls: true
            ,
            noCode: true, onDemand: true,
            discardSelector: ".discard-answer"
            ,immediatelyShowMarkdownHelp:true
            );



            );













            draft saved

            draft discarded


















            StackExchange.ready(
            function ()
            StackExchange.openid.initPostLogin('.new-post-login', 'https%3a%2f%2fmath.stackexchange.com%2fquestions%2f2661300%2fprove-uniform-continuity-of-the-composition-of-two-uniformly-continuous-function%23new-answer', 'question_page');

            );

            Post as a guest















            Required, but never shown

























            1 Answer
            1






            active

            oldest

            votes








            1 Answer
            1






            active

            oldest

            votes









            active

            oldest

            votes






            active

            oldest

            votes









            3












            $begingroup$

            This is the right idea but what you have written is nonsense if taken literally. It is extremely important to clearly state the hypotheses and quantifiers in each statement you make. For instance, we are not "given" $$left|x' - y' right| < delta_f implies left| fleft(x'right) - fleft(y'right) right| < epsilon.$$ Rather, we are given that there exists $delta_f>0$ such that for all $x',y'inmathbbR$, this implication is true. Clean up all your statements similarly and you'll have a great proof.






            share|cite|improve this answer









            $endgroup$

















              3












              $begingroup$

              This is the right idea but what you have written is nonsense if taken literally. It is extremely important to clearly state the hypotheses and quantifiers in each statement you make. For instance, we are not "given" $$left|x' - y' right| < delta_f implies left| fleft(x'right) - fleft(y'right) right| < epsilon.$$ Rather, we are given that there exists $delta_f>0$ such that for all $x',y'inmathbbR$, this implication is true. Clean up all your statements similarly and you'll have a great proof.






              share|cite|improve this answer









              $endgroup$















                3












                3








                3





                $begingroup$

                This is the right idea but what you have written is nonsense if taken literally. It is extremely important to clearly state the hypotheses and quantifiers in each statement you make. For instance, we are not "given" $$left|x' - y' right| < delta_f implies left| fleft(x'right) - fleft(y'right) right| < epsilon.$$ Rather, we are given that there exists $delta_f>0$ such that for all $x',y'inmathbbR$, this implication is true. Clean up all your statements similarly and you'll have a great proof.






                share|cite|improve this answer









                $endgroup$



                This is the right idea but what you have written is nonsense if taken literally. It is extremely important to clearly state the hypotheses and quantifiers in each statement you make. For instance, we are not "given" $$left|x' - y' right| < delta_f implies left| fleft(x'right) - fleft(y'right) right| < epsilon.$$ Rather, we are given that there exists $delta_f>0$ such that for all $x',y'inmathbbR$, this implication is true. Clean up all your statements similarly and you'll have a great proof.







                share|cite|improve this answer












                share|cite|improve this answer



                share|cite|improve this answer










                answered Feb 22 '18 at 4:26









                Eric WofseyEric Wofsey

                193k14221352




                193k14221352



























                    draft saved

                    draft discarded
















































                    Thanks for contributing an answer to Mathematics Stack Exchange!


                    • Please be sure to answer the question. Provide details and share your research!

                    But avoid


                    • Asking for help, clarification, or responding to other answers.

                    • Making statements based on opinion; back them up with references or personal experience.

                    Use MathJax to format equations. MathJax reference.


                    To learn more, see our tips on writing great answers.




                    draft saved


                    draft discarded














                    StackExchange.ready(
                    function ()
                    StackExchange.openid.initPostLogin('.new-post-login', 'https%3a%2f%2fmath.stackexchange.com%2fquestions%2f2661300%2fprove-uniform-continuity-of-the-composition-of-two-uniformly-continuous-function%23new-answer', 'question_page');

                    );

                    Post as a guest















                    Required, but never shown





















































                    Required, but never shown














                    Required, but never shown












                    Required, but never shown







                    Required, but never shown

































                    Required, but never shown














                    Required, but never shown












                    Required, but never shown







                    Required, but never shown







                    Popular posts from this blog

                    Triangular numbers and gcdProving sum of a set is $0 pmod n$ if $n$ is odd, or $fracn2 pmod n$ if $n$ is even?Is greatest common divisor of two numbers really their smallest linear combination?GCD, LCM RelationshipProve a set of nonnegative integers with greatest common divisor 1 and closed under addition has all but finite many nonnegative integers.all pairs of a and b in an equation containing gcdTriangular Numbers Modulo $k$ - Hit All Values?Understanding the Existence and Uniqueness of the GCDGCD and LCM with logical symbolsThe greatest common divisor of two positive integers less than 100 is equal to 3. Their least common multiple is twelve times one of the integers.Suppose that for all integers $x$, $x|a$ and $x|b$ if and only if $x|c$. Then $c = gcd(a,b)$Which is the gcd of 2 numbers which are multiplied and the result is 600000?

                    Ingelân Ynhâld Etymology | Geografy | Skiednis | Polityk en bestjoer | Ekonomy | Demografy | Kultuer | Klimaat | Sjoch ek | Keppelings om utens | Boarnen, noaten en referinsjes Navigaasjemenuwww.gov.ukOffisjele webside fan it regear fan it Feriene KeninkrykOffisjele webside fan it Britske FerkearsburoNederlânsktalige ynformaasje fan it Britske FerkearsburoOffisjele webside fan English Heritage, de organisaasje dy't him ynset foar it behâld fan it Ingelske kultuergoedYnwennertallen fan alle Britske stêden út 'e folkstelling fan 2011Notes en References, op dizze sideEngland

                    Հադիս Բովանդակություն Անվանում և նշանակություն | Դասակարգում | Աղբյուրներ | Նավարկման ցանկ